- PowerScore Staff
- Posts: 5972
- Joined: Mar 25, 2011
- Sun Jun 08, 2014 10:19 am
#87685
Complete Question Explanation
(The complete setup for this game can be found here: lsat/viewtopic.php?f=151&t=8310)
The correct answer choice is (B)
From the analysis made during the setup, we know that Q must be last, and therefore answer choice (B) is correct. However, even if you do not draw this inference, this question can be solved easily by referring to prior hypotheticals:
The question stem to question #15 indicates that P does not have to depart first, eliminating answer choice (A).
The question stem to question #17 indicates that S can depart third, eliminating answer choice (D).
(The complete setup for this game can be found here: lsat/viewtopic.php?f=151&t=8310)
The correct answer choice is (B)
From the analysis made during the setup, we know that Q must be last, and therefore answer choice (B) is correct. However, even if you do not draw this inference, this question can be solved easily by referring to prior hypotheticals:
The question stem to question #15 indicates that P does not have to depart first, eliminating answer choice (A).
The question stem to question #17 indicates that S can depart third, eliminating answer choice (D).
Dave Killoran
PowerScore Test Preparation
Follow me on X/Twitter at http://twitter.com/DaveKilloran
My LSAT Articles: http://blog.powerscore.com/lsat/author/dave-killoran
PowerScore Podcast: http://www.powerscore.com/lsat/podcast/
PowerScore Test Preparation
Follow me on X/Twitter at http://twitter.com/DaveKilloran
My LSAT Articles: http://blog.powerscore.com/lsat/author/dave-killoran
PowerScore Podcast: http://www.powerscore.com/lsat/podcast/